17
$\begingroup$

In this topic, I will use the word uncountable group referring to groups whose cardinality is $\leq|\mathbb R|$.

Notation: $R$ is the hyperfinite $II_1$-factor, $\omega$ is a free ultrafilter on the natural numbers, $R^\omega$ is the tracial ultrapower, $\tau$ is the unique normalized trace on $R^\omega$, $U(R^\omega)$ is the unitary group of $R^\omega$.

Definition: A group $G$ is called hyperlinear if there is a group monomorphism $\theta:G\rightarrow U(R^\omega)$ such that $\tau(\theta(g))=0$ for all $g\neq1$.

Question 1: Does there exist an uncountable non-hyperlinear group?

A bit of background: the same question for countable groups is known as Connes' embedding problem for groups and it's still unsolved. When I began my PhD (Nov. 2008), my former advisor Florin Radulescu told L. Paunescu and myself that he would have liked to have a better understanding of the problem in higher cardinality. In particular, he asked us to see whether the free group on uncountable many generators, here denoted by $\mathbb F_c$, and the circle group $S_1$ were hyperlinear. Maybe he expected that one of them was not, but we came up with the general result that every subrgroup of $U(R^\omega)$ is hyperlinear (this is basically proved in http://arxiv.org/abs/0911.4978); in particular $S_1$ is hyperlinear and, with an additional 10-line argument, also $\mathbb F_c$ turns out to be hyperlinear. Moreover, the result is quite general and excludes many possible example a priori, making Question 1, in my modest opinion, non-trivial and interesting. At some point, maybe also talking with someone else (but I don't remember exactly who), I got quite convinced that Question 1 has the same level of difficulty of Connes' problem and that they may be actually equivalent. In this case, I would like to find a formal way to express that. In this view, a positive answer to the following question would not completely solve the problem, but would be a nice starting point.

Question 2: Let $G$ be an uncountable group. Do there always exist countable groups $G_1,G_2,\ldots$ and a free ultrafilter $\omega$ such that $G$ embeds into the algebraic ultraproduct of the $G_i$'s?

Update: as shown my Simon Thomas below, the answer is positive assuming CH. On the other hand, Joel's answer shows that without CH we have some weaker result. For instance, Question 2 has affirmative answer if we allow the sequence $G_i$ to be indexed by a possibly non-countable set $I$.

Thanks in advance,

Valerio

$\endgroup$
5
  • $\begingroup$ Don't you mean that uncountable groups have cardinality $\geq |\mathbb{R}|$ (instead of $\leq |\mathbb{R}|$) ? $\endgroup$
    – Max Horn
    Nov 6, 2011 at 16:10
  • 4
    $\begingroup$ No, it's right as I wrote. If were $\geq|\mathbb R|$, the answer would have been trivial: take a big enough group. Indeed I am using this non-standard terminology just to avoid easy counterexamples. $\endgroup$ Nov 6, 2011 at 16:21
  • 3
    $\begingroup$ Since any group is a metric group with the trivial distance $d(g,h)=1-\delta_{gh}$, it seems to me that your question 2 asks whether every ``uncountable'' group embeds into the algebraic ultraproduct of countable groups. I'm sure that model-theorists know about that... $\endgroup$ Nov 6, 2011 at 21:30
  • 1
    $\begingroup$ Alain, at the beginning I wrote basically what you have now written, but (fault of my lack of knowledge) it was not clear to me what is a purely algebraic ultraproduct of countable groups. What is the equivalence relation we mod out with? Maybe it's just: two sequence in the cartesian product are equal if they coincide on an element of $\omega$.. $\endgroup$ Nov 6, 2011 at 21:50
  • 4
    $\begingroup$ @Valerio: what else? (:-) $\endgroup$ Nov 6, 2011 at 21:54

2 Answers 2

11
$\begingroup$

The general situation, where CH fails, may be informed by the Keisler-Shelah isomorphism theorem, which asserts that two first-order structures have isomorphic ultrapowers if and only if they have the same first-order theory.

In particular, for any infinite group $G$ at all, of any size, we may take a countable elementary subgroup $H$, meaning in particular that they have the same first-order theory, and so there is a nonprincipal ultrafilter $U$ on an index set $I$ such that the ultrapowers $G^I/U\cong H^I/U$ are isomorphic. Since every first-order structure maps elementarily into its ultrapowers, this means in particular that $G$ maps elementarily (and hence monomorphically) into an ultrapower of $H$, a countable group.

Thus, this fully answers the version of question 2 in which we allow the ultrafilter to live on a bigger index set:

Theorem. For every group $G$ there is a countable group $H$ and a free ultrafilter $U$ on a set, such that $G$ embeds into the ultrapower $H^I/U$.

If you want to insist that the ultrafilter concentrate on index set $\mathbb{N}$, however, then things become more complicated. If the CH holds, then the Keisler-Shelah theorem shows that any two groups of size at most $2^{\aleph_0}$ and with the same theory have isomorphic ultrapowers by an ultrafilter on $\aleph_0$, and so the desired result is attained. In the non-CH case, however, what we seem to get is that for any cardinal $\lambda$, if $\beta$ is smallest such that $\lambda^\beta\gt\lambda$, then any two groups of size $\beta$ with the same theory have isomorphic utrapowers using an ultrafilter on $\lambda$. Thus, they each map into an ultrapower of the other.

The Keisler-Shelah theorem was proved first by Keisler in the case that GCH holds, using saturation ideas as in Simon's answer. The need for the GCH was later removed by Shelah.

$\endgroup$
13
$\begingroup$

A partial answer to Alain's question ... Suppose that the Continuum Hypothesis $CH$ holds. Let $G$ be any group of size $2^{\aleph_{0}} = \aleph_{1}$. Then there exists a countable subgroup $H$ of $G$ with the same first order theory $T$. Let $\Gamma = \prod_{\mathcal{U}} H_{n}$ be the ultraproduct such that every $H_{n} = H$. Then $CH$ implies that $\Gamma$ is a saturated model of $T$ and hence $G$ embeds into $\Gamma$.

Your question concerning the possibility of embedding groups $G$ with $|G| \leq 2^{\aleph_{0}}$ into ultraproducts $\prod_{\mathcal{U}} H_{n}$ over a countable index set might be interesting in the case when $CH$ fails. There is a similar open problem concerning sofic groups. It is an easily seen that every group is sofic iff $Sym(\mathbb{N})$ embeds in some universal sofic group. However, even if every group is sofic, it is not clear whether or not $Sym(\mathbb{N})$ embeds in a universal sofic group arising from an ultrafilter over $\mathbb{N}$.

If you are interested in embeddings using ultraproducts over larger index sets, then you can make use of an ancient result of Malcev, which says that every group $G$ embeds into a suitable ultraproduct of its finitely generated subgroups. Here the index set $I$ is the set of its finitely generated subgroups, which is uncountable if $G$ is uncountable.

PS: In Shelah's paper on ultraproducts, he mentions that if Martin's Axiom holds, then there exists an ultrafilter $\mathcal{D}$ over $\mathbb{N}$ such that for every countable structure $M$ for a countable language, the corresponding ultraproduct $\prod_{\mathcal{D}}M$ is saturated. In particular, it is consistent with the failure of $CH$ that every group $G$ with $|G| \leq 2^{\aleph_{0}}$ embeds into an ultraproduct $\prod_{\mathcal{U}} H_{n}$ over a countable index set.

$\endgroup$
7
  • 2
    $\begingroup$ Except I am told that I am now a set theorist. So naturally I hope that your question is independent of ZFC. $\endgroup$ Nov 6, 2011 at 22:06
  • 1
    $\begingroup$ @Simon: Oops... But won't you attend the conference at CIRM, two weeks from now? $\endgroup$ Nov 6, 2011 at 22:11
  • 2
    $\begingroup$ @Alain: The organizers are very broad minded. @Valerio: The only reason that I know that a countable number of copies works is a "soft" model-theoretic reason. Assuming CH, the ultraproduct of countably many copies of a countable structure is saturated and this implies that any other structure with the same first order theory must embed into it. $\endgroup$ Nov 6, 2011 at 22:49
  • 1
    $\begingroup$ Simon, many thanks for your answer + clarification. I have accepted Joel's answer only because he considers also the case when CH may fail. $\endgroup$ Nov 7, 2011 at 10:45
  • 2
    $\begingroup$ Nice observation in the P.S., Simon! $\endgroup$ Nov 7, 2011 at 14:32

Your Answer

By clicking “Post Your Answer”, you agree to our terms of service and acknowledge you have read our privacy policy.

Not the answer you're looking for? Browse other questions tagged or ask your own question.